J'ai un problème avec les ensembles canoniques
Répondre à la discussion
Page 1 sur 2 1 DernièreDernière
Affichage des résultats 1 à 30 sur 45

J'ai un problème avec les ensembles canoniques



  1. #1
    gatsu

    J'ai un problème avec les ensembles canoniques


    ------

    Bonjour,

    Je précise d'avance pour tous ceux qui en douteraient que je sais à peu près ce qu'est la mécanique statistique (MS) d'équilibre et que je l'utilise tous les jours.
    C'est notamment parce que je l'utilise fréquemment que je m'intéresse de très très près à ses fondements.
    Lorsqu'on lie et étudie les ouvrages et papiers sur les fondements de la MS on remarque assez vite que quel que soit le point de vue adopté (point de vue dynamique ou inférence statistique ou mélange des deux) pour justifier la MS d'équilibre, seule la distribution de l'ensemble microcanonique est "démontrable".
    Les autres ensembles statistiques "d'équilibre" sont souvent justifiés a postériori via l'équivalence d'ensembles ou dans des situations spécifiques considérant un sous système d'un grand système régit par la statistique microcanonique.

    Pourtant il me semble qu'il y a un problème avec l'ensemble microcanonique. En effet il semble être plus qu'un ensemble d'équilibre d'après ce que j'en ai compris.
    Un truc notamment que je ne comprends pas c'est que l'ensemble microcanonique contient des microétats qui ne sont pas des microétats correspondants à des macroétats d'équilibre (je pense en particulier aux microétats pour lesquels toutes les particules d'un gaz sont toutes réunies dans un petit volume d'une enceinte plus grande).
    Preuve en est que assez usuellement, on peut avoir une estimation de la probabilité d'occurence d'un microétat correspondant à un macroétat hors d'équilibre en appliquant le principe d'équiprobabilité des microétats (ensemble microcanonique) et en faisant le rapport

    .

    Ma question est donc, dans ce contexte, pourquoi est ce lorsqu'on "dérive" (ou justifie) l'ensemble microcanonique (via une inférence statistique ou via le theorème de Birkhoff par exemple) on dit directement que l'ensemble microcanonique correspond à une distribution d'équilibre alors que ça ne semble visiblement pas être rigoureusement le cas.

    Pour ceux que ça intéresse j'avais ouvert un fil (de façon un peut differente)sur Physicsforum ici (je ne sais pas si j'ai le droit de faire ça).

    Merci d'avance pour vos remarques et suggestions !

    -----
    "Au fond..la musique si on la prend note par note c'est assez nul". Geluck

  2. #2
    mariposa

    Re : J'ai un problème avec les ensembles canoniques

    Citation Envoyé par gatsu Voir le message
    Lorsqu'on lie et étudie les ouvrages et papiers sur les fondements de la MS on remarque assez vite que quel que soit le point de vue adopté (point de vue dynamique ou inférence statistique ou mélange des deux) pour justifier la MS d'équilibre, seule la distribution de l'ensemble microcanonique est "démontrable".
    .
    Bonjour,

    Pour moi, dès lors que l'on parle probabilité, il y a un modèle mathématique dont la validité est d'abord justifié par les conséquences. Ce qui n'empèche pas par ailleurs de remonter en amont pour trouver des fondements (un problème non résolu pour que je saches).
    .
    Pour ce qui est de l'ensemble microcanonique: Pour un système isolé à l'équilibre thermodynamique ( donc une énergie E fixée) tous les états quantiques accessibles sont équiprobables. en conséquences.

    P(i) = 1/D(Ei)

    où D(Ei) est la densité d'états de l'énergie Ei

    Ceci définit l'ensemble microcanique. Je le considère comme équivalent à l'hypothèse des dés ou l'on considère que les états accessibles sont équiprobables.



    Pourtant il me semble qu'il y a un problème avec l'ensemble microcanonique. En effet il semble être plus qu'un ensemble d'équilibre d'après ce que j'en ai compris.
    Un truc notamment que je ne comprends pas c'est que l'ensemble microcanonique contient des microétats qui ne sont pas des microétats correspondants à des macroétats d'équilibre (je pense en particulier aux microétats pour lesquels toutes les particules d'un gaz sont toutes réunies dans un petit volume d'une enceinte plus grande).
    Preuve en est que assez usuellement, on peut avoir une estimation de la probabilité d'occurence d'un microétat correspondant à un macroétat hors d'équilibre en appliquant le principe d'équiprobabilité des microétats (ensemble microcanonique) et en faisant le rapport

    .
    Supposons que tu as 2 particules dans une boite que tu divises en 2 par la pensée.

    La probabilité d'avoir 2 particules a gauche est 1/4
    La probabilité d'avoir 2 particules a droite est 1/4
    la probabilité d'avoir une particule a gauche et une particule a droite est 1/2
    .
    On voit a travers cet exemple que la probabilité est plus forte d'avoir 1 particule de chaque coté parce que la densité d'état est 2.
    .
    Il est facile de comprendre que pour N particules (N grand) la probabilité d'avoir moitié-moitié est infiniment plus probable parce que la densité d'état est très élevèe comparée a celle ou toutes les particules sont du même coté.
    .
    Donc pour un système microcanonique, la configuration est très piquée en probabilité et d'autant que N est élevé.

  3. #3
    gatsu

    Re : J'ai un problème avec les ensembles canoniques

    Citation Envoyé par mariposa Voir le message
    .

    Pour ce qui est de l'ensemble microcanonique: Pour un système isolé à l'équilibre thermodynamique ( donc une énergie E fixée)
    Précisément non ce n'est pas parce que l'energie est fixée que le système est à l'équilibre. N'importe quel système avec une dynamique hamiltonienne (que ce soit quantique ou classique) a une energie fixée et il n'est pas pour autant à l'équilibre (c'est à dire qu'il n'est pas forcément dans une configuration microscopique correspondant à l'état macroscopique le plus "peuplé" en microétats).

    Ceci définit l'ensemble microcanique. Je le considère comme équivalent à l'hypothèse des dés ou l'on considère que les états accessibles sont équiprobables.
    Moi aussi

    Supposons que tu as 2 particules dans une boite que tu divises en 2 par la pensée.

    La probabilité d'avoir 2 particules a gauche est 1/4
    La probabilité d'avoir 2 particules a droite est 1/4
    la probabilité d'avoir une particule a gauche et une particule a droite est 1/2
    .
    On voit a travers cet exemple que la probabilité est plus forte d'avoir 1 particule de chaque coté parce que la densité d'état est 2.
    .
    Il est facile de comprendre que pour N particules (N grand) la probabilité d'avoir moitié-moitié est infiniment plus probable parce que la densité d'état est très élevèe comparée a celle ou toutes les particules sont du même coté.
    .
    Donc pour un système microcanonique, la configuration est très piquée en probabilité et d'autant que N est élevé.
    Oui et alors ? Est ce que ça te semble normal à toi qu'une distribution d'équilibre puisse te donner des renseignements sur des états hors équilibre (j'utilise ici une définition de l'equilbre pour laquelle notamment la densité d'un gaz est uniforme en absence de champ exterieur).
    "Au fond..la musique si on la prend note par note c'est assez nul". Geluck

  4. #4
    mariposa

    Re : J'ai un problème avec les ensembles canoniques

    Citation Envoyé par gatsu Voir le message
    Précisément non ce n'est pas parce que l'energie est fixée que le système est à l'équilibre. N'importe quel système avec une dynamique hamiltonienne (que ce soit quantique ou classique) a une energie fixée et il n'est pas pour autant à l'équilibre (c'est à dire qu'il n'est pas forcément dans une configuration microscopique correspondant à l'état macroscopique le plus "peuplé" en microétats).
    .
    Effectivement le fait qu'un système isolé possède une énergie fixée n'est en rien synonyme d'équilibre thermodynamique. ce qui définit l'équilibre thermodynamique c''est l'équiprobabilité des états accessibles.
    .
    Soit un système hors d'équilibre à l'instant t= 0.
    .
    Dans ce cas les probabilité d'occupation des étas accessibles s'écrivent:

    P(i,t) formule qui indique a la fois que les probabilités évoluent avec le temps (ce qui n'est pas vrai à l'équilibre) et qu'elles ne sont pas égales ( elles sont égales à l'équilibre).

    A l'équilibre: P(i,infini) = 1/D(Ei)

    Une façon est de visualiser ceci comme un processus de Markov à temps discret. A t=0 tu as une répartition des états quelconque (système hors d'équilibre. Cette répartition évolue et converge vers une distribution uniforme au bout d'un temps infini et ceci quelquesoit l'état de départ.
    .
    Pour reprendre ton exemple: Mettons le système hors d'équilibre avec toutes les N particules as a gauche à t=0. Un certain instant plus tard la probabilité d'avoir N-1 particules a gauche et 1 particule a droite est élevée. Au bout d'un temps infini le système est N/2, N/2 et la probabilité d'avoir N particules a gauche est ridiculement faible alors qu'elle était voisine de 1 quelques temps apres zéro.

    Autrement dit du point de vue etat: toutes les particules a gauches: la probabilité est forte pour le système hors d'équilibre et quasi-nulle pour le système à l'équilibre.
    ;
    Autrement dit pour un même état (ici même configuration) les probabilités different entre équilibre et non équilibre thermodynamique.

  5. A voir en vidéo sur Futura
  6. #5
    gatsu

    Re : J'ai un problème avec les ensembles canoniques

    Citation Envoyé par mariposa Voir le message
    .
    ce qui définit l'équilibre thermodynamique c''est l'équiprobabilité des états accessibles.
    Je ne vois personnellement pas comment on peut définir l'équilibre comme ça a priori.
    Encore une fois, lorsqu'on construit la distribution microcanonique (E,V,N) c'est la seule distribution de proba. "raisonnable" (point de vue Bayesien) qu'on puisse construire compte tenu des seules informations qu'on a sur le système à tout instant (justement E,V et N).

    Par ailleurs, l'équilibre est plutot défini comme étant le macroétat (i.e. d'équilibre) qui correspond aux plus de microétats mais c'est tout.


    Soit un système hors d'équilibre à l'instant t= 0.
    .
    Dans ce cas les probabilité d'occupation des étas accessibles s'écrivent:

    P(i,t) formule qui indique a la fois que les probabilités évoluent avec le temps (ce qui n'est pas vrai à l'équilibre) et qu'elles ne sont pas égales ( elles sont égales à l'équilibre).

    A l'équilibre: P(i,infini) = 1/D(Ei)
    C'est là où je souhaite faire une nuance. La limite du temps infini a toujours été (et est encore) un problème dans les fondements de la MS. En toute rigueur, comme l'avait objecté Zermelo à la fin de XIXe on va forcément finir par retomber sur un macroétat (en vertu du principe de récurence de Poincaré...même si je ne sais pas si il existe un équivalent en MQ) hors d'équilibre à un moment ou un autre.
    Cette distinction est importante à mon sens car lorsqu'on construit l'ensemble microcanonique en imaginant que le système a attendu un temps infini on le laisse passer par tous les états macroscopiques possibles (qu'ils soient d'équilibre ou hors équilibre) et c'est donc naturellement qu'on se retrouve avec une distribution "d'équilibre" qui contient des microétats hors equilibre (ce que je trouve bizarre fondamentalement pour un ensemble statistique "d'equilibre").

    Une façon est de visualiser ceci comme un processus de Markov à temps discret. A t=0 tu as une répartition des états quelconque (système hors d'équilibre. Cette répartition évolue et converge vers une distribution uniforme au bout d'un temps infini et ceci quelquesoit l'état de départ.
    Le biais qui intervient dans le choix de la direction du temps (et qui consiste à ne pas discriminer temps long et temps infini noatemment) est justement relié à l'hypothèse de chaos moléculaire (ce qui revient en discret à un processus de Markov).

    Pour reprendre ton exemple: Mettons le système hors d'équilibre avec toutes les N particules as a gauche à t=0. Un certain instant plus tard la probabilité d'avoir N-1 particules a gauche et 1 particule a droite est élevée. Au bout d'un temps infini le système est N/2, N/2 et la probabilité d'avoir N particules a gauche est ridiculement faible alors qu'elle était voisine de 1 quelques temps apres zéro.
    Je suis d'accord mais ce qui est embetant pour moi d'un point de vue fondamental c'est que ce calcul se fait précisément dans le cadre d'une statistique microcanonique.

    Autrement dit du point de vue etat: toutes les particules a gauches: la probabilité est forte pour le système hors d'équilibre et quasi-nulle pour le système à l'équilibre.
    Autrement dit pour un même état (ici même configuration) les probabilités different entre équilibre et non équilibre thermodynamique.
    Je ne suis pas sûr d'être d'accord sur ce dernier point. Les probabilités sont utilisées usuellement pour "parier" sur le microétat qui correspond à un macroétat observé et non pas l'inverse il me semble.
    "Au fond..la musique si on la prend note par note c'est assez nul". Geluck

  7. #6
    mariposa

    Re : J'ai un problème avec les ensembles canoniques

    Citation Envoyé par gatsu Voir le message
    Je ne vois personnellement pas comment on peut définir l'équilibre comme ça a priori.
    ;
    C'est la définition officielle de l'équilibre thermodynamique dont les conséquences sont vérifiées sans problème. Si on veut une explication amont de cet hypothèse cela rncoie au problème de l'ergocité qui n'est démontrée que dans quelques particuliers (travaux de Sinaï et al..)

    Par ailleurs, l'équilibre est plutot défini comme étant le macroétat (i.e. d'équilibre) qui correspond aux plus de microétats mais c'est tout.
    .C'est une écriture incomplète et maladoite de l'équilibre thermodynamique pour la simple raison que tu ne peux rien dérouler.


    C'est là où je souhaite faire une nuance. La limite du temps infini a toujours été (et est encore) un problème dans les fondements de la MS. En toute rigueur, comme l'avait objecté Zermelo à la fin de XIXe on va forcément finir par retomber sur un macroétat (en vertu du principe de récurence de Poincaré...même si je ne sais pas si il existe un équivalent en MQ) hors d'équilibre à un moment ou un autre.
    .
    C'est une problématique purement théorique. Il est évidemment qu'un système en physique classique est representée par une trajectoire dans un espace de phase. Ce qui veut dire qu'un système reviendra au bout d'un certain temps dans un état voisin de l'état initial. donc un système thermodynamique loin de l'équilibre reviendra dans le même état au bout d'un certain temps. Ce certain temps vaut des milliards de fois la durée de notre univers! A contrarion la plupart des systèmes reviennent à l"équilibre en moins de 1s voire une µseconde.


    Le biais qui intervient dans le choix de la direction du temps (et qui consiste à ne pas discriminer temps long et temps infini noatemment) est justement relié à l'hypothèse de chaos moléculaire (ce qui revient en discret à un processus de Markov).
    On ne choisit pas le temps, ce sont les systèmes physiques qui imposent la direction du temps. Les systèmes isolés évoluent vers l'équilibre thermodynamique caractérisé par croissance d'entropie. (voir les travaux de Prigogine)



    Je ne suis pas sûr d'être d'accord sur ce dernier point. Les probabilités sont utilisées usuellement pour "parier" sur le microétat qui correspond à un macroétat observé et non pas l'inverse il me semble.
    .
    Les probabilités servent a calculerb les valeurs moyennes observées expérimentalement

    <A> = Ai.P(i)

    avec p(i) la probabilité que le système soit dans un état i.

  8. #7
    gatsu

    Re : J'ai un problème avec les ensembles canoniques

    Citation Envoyé par mariposa Voir le message
    ;
    C'est la définition officielle de l'équilibre thermodynamique dont les conséquences sont vérifiées sans problème. Si on veut une explication amont de cet hypothèse cela rncoie au problème de l'ergocité qui n'est démontrée que dans quelques particuliers (travaux de Sinaï et al..)
    Je serais assez curieux de savoir où est ce que tu as vu que c'était une définition officielle. C'est par contre une définition rencontrée assez souvent qui souligne pour moi le fait qu'il y a plusieurs points de vue sur le sujet. Comme je l'ai déjà dit je pense qu'il doit y avoir une difference, au moins de principe, entre l'équilibre et le temps infini (qui est indisociable de l'ergodicité même au sens de Birkhoff).

    .
    C'est une écriture incomplète et maladoite de l'équilibre thermodynamique pour la simple raison que tu ne peux rien dérouler.
    Rien dérouler ? What do you mean ?
    C'est pourtant une autre défintion de l'equilibre thermodynamique que j'estime personnellement être la bonne car d'avantage reliée (ou reliable) aux résultats de manips.

    C'est une problématique purement théorique. Il est évident qu'un système en physique classique est representée par une trajectoire dans un espace de phase. Ce qui veut dire qu'un système reviendra au bout d'un certain temps dans un état voisin de l'état initial. donc un système thermodynamique loin de l'équilibre reviendra dans le même état au bout d'un certain temps. Ce certain temps vaut des milliards de fois la durée de notre univers! A contrario la plupart des systèmes reviennent à l"équilibre en moins de 1s voire une µseconde.
    Comme je l'ai déjà dit dans le fil de Physicsforum que j'ai mis en lien (et que je vous invitais à lire) je ne cherche pas vraiment à savoir pourquoi ça marche en pratique car je le sais déjà et Boltzmann avait déjà à peu près correctement répondu à toutes ces questions "pratiques" il y a plus d'un siècle. Mon problème est plus de principe et "fondamental" je dirais. Le truc c'est que grace à la loi des grands nombres et au theorème central limite, on peut faire à peu près tout ce que l'on veut en statistique d'équilibre et ça marchera. L'idée est de savoir ce qu'il se passe si il n'y a pas 10^23 degrés de libertés mais seulement une centaine par exemple (ou même moins qui sait).
    Par exemple "ta" défintion de l'équilibre découle naturellement de la "mienne" puisqu'on peut montrer que pour des objets macroscopiques (nombre d'Avogadro), la mesure du volume de l'espace des phases correspondant à des macroétats hors d'équilibre tend vers zero.
    Du coup j'ai l'impression que souvent on mélange la cause et l'effet, ce qui est une erreur logique acceptable lorsqu'on est à la limite thermo et tout le tralala mais qui devient bien plus délicat lorsqu'on se ballade à la limite du domaine de validité de la MS (encore inconnu à ce jour il me semble).

    On ne choisit pas le temps, ce sont les systèmes physiques qui imposent la direction du temps. Les systèmes isolés évoluent vers l'équilibre thermodynamique caractérisé par croissance d'entropie. (voir les travaux de Prigogine)
    On "choisit" (a priori encore une fois) le temps en faisant un coarse graining du système (i.e. en observant l'évolution de macrovariables). Si on restait à une résolution trop petite avec une connaissance parfaite du système à un instant donné, il n'y aurait pas de second principe.
    L'équation de Boltzmann par exemple est directement non invariante par renversement du temps. Ceci est dû au fait qu'une hypothèse a été introduite de façon had hoc dans la dynamique des collisions à deux corps

    Les probabilités servent a calculerb les valeurs moyennes observées expérimentalement
    <A> = Ai.P(i)
    avec p(i) la probabilité que le système soit dans un état i.
    Oui mais elles sont constraintes par des valeurs de macrovariables plus ou moins en rapport avec la notion d'equilibre (E,V,N par exemple ou <E>,V,N etc...).
    "Au fond..la musique si on la prend note par note c'est assez nul". Geluck

  9. #8
    mariposa

    Re : J'ai un problème avec les ensembles canoniques

    Citation Envoyé par gatsu Voir le message

    On "choisit" (a priori encore une fois) le temps en faisant un coarse graining du système (i.e. en observant l'évolution de macrovariables). Si on restait à une résolution trop petite avec une connaissance parfaite du système à un instant donné, il n'y aurait pas de second principe.
    L'équation de Boltzmann par exemple est directement non invariante par renversement du temps. Ceci est dû au fait qu'une hypothèse a été introduite de façon had hoc dans la dynamique des collisions à deux corps
    le rapport entre l'invariance par renversement du temps de la MC et de l'évolution unidirectionnelle donnée par l'équationde Boltzmann est bien connu. Il s'agit d'un phénomène d'émergence qui résulte du grand nombre de particules.
    .
    Le problème est bien connu et a été résolu par Bogolioubiov. La seule hypothèse est un principe d'afaiblissement spatiale des corrélations.
    .
    Il s'agit de partir de la fonction de distribution excate d'un système à N particules. On peut à partir de cela développer une suite enchainée de fonctions de distributions à 1 corps, 2 corps, 3 corps...etc... C'est que l'on apperlle la hierarchie BBGKY
    .
    On montre que pour un temps t supérieur a quelques collisions, toutes les fonctions de distributions à plusieurs particules sont des fonctionnelles de la fonction de distribution à 1 particule. On peut calculer ainsi l'évolution de la fonction de distribution à 1 corps df/dt = .... dont l'équation de Bolzmann est une simplification

  10. #9
    gatsu

    Re : J'ai un problème avec les ensembles canoniques

    Citation Envoyé par mariposa Voir le message
    le rapport entre l'invariance par renversement du temps de la MC et de l'évolution unidirectionnelle donnée par l'équationde Boltzmann est bien connu. Il s'agit d'un phénomène d'émergence qui résulte du grand nombre de particules.
    Ca ne change rien à ce que j'ai dit. Peut importe le nombre de particules, si on regarde trop précisément on ne verra jamais l'évolution recherchée. Dit autrement la propriété émergente n'est visible que lorsque tu te mets à l'échelle pour la voir.

    Le problème est bien connu et a été résolu par Bogolioubiov. La seule hypothèse est un principe d'afaiblissement spatiale des corrélations.
    .
    Il s'agit de partir de la fonction de distribution excate d'un système à N particules. On peut à partir de cela développer une suite enchainée de fonctions de distributions à 1 corps, 2 corps, 3 corps...etc... C'est que l'on apperlle la hierarchie BBGKY
    Je connais dans le principe la hierarchie BBGKY (puisqu'utilisée en physique des liquides) et elle ne permet pas a priori d'expliquer l'irreversibilité macroscopique.
    Lorsqu'on s'interesse à l'ordre 1 de la hierarchie on a un truc du genre

    est une fonctionnelle.
    Ce qu'il se passe c'est que Boltzmann a donné de façon phénomènologique (et un peu had hoc) une forme spécifique à cette fonctionnelle (basée sur l'hypothèse de chaos moléculaire) qui a induit demblé une irreversibilité dans l'equation d'évolution de la distribution à une particule.
    "Au fond..la musique si on la prend note par note c'est assez nul". Geluck

  11. #10
    gatsu

    Re : J'ai un problème avec les ensembles canoniques

    Je me permets un petit up en esperant qu'il n'y a pas que deux personnes interessées par cette question...
    "Au fond..la musique si on la prend note par note c'est assez nul". Geluck

  12. #11
    invite1c3dc18e

    Re : J'ai un problème avec les ensembles canoniques

    Citation Envoyé par gatsu Voir le message
    Mon problème est plus de principe et "fondamental" je dirais. Le truc c'est que grace à la loi des grands nombres et au theorème central limite, on peut faire à peu près tout ce que l'on veut en statistique d'équilibre et ça marchera. L'idée est de savoir ce qu'il se passe si il n'y a pas 10^23 degrés de libertés mais seulement une centaine par exemple (ou même moins qui sait).
    pas mal de travaux ont été effectués où sont en cours ce sujet. On montre que la position d'équilibre ne peut être atteinte pour un ensemble trop petit de particules. Typiquement en dessous d'une centaine justement.

    Je t'invite à lire les travaux de P. Gaspard à ce sujet

    par exemple:
    http://scitation.aip.org/getpdf/serv...ps&prog=normal


    Cordialement,

    Anacarsis

  13. #12
    mariposa

    Re : J'ai un problème avec les ensembles canoniques

    Citation Envoyé par gatsu Voir le message
    Ca ne change rien à ce que j'ai dit. Peut importe le nombre de particules, si on regarde trop précisément on ne verra jamais l'évolution recherchée. Dit autrement la propriété émergente n'est visible que lorsque tu te mets à l'échelle pour la voir.
    .
    ta phrase est contradictoire en elle-même. Le mécanisme émergent est une propriété nouvelle qui apparait seulement pour un grand nombre de particules. Donc il ne faut pas surtout écrire: "peu importe le nombre de particules".

    Pour que les lois statistiques émergent il faut justement un grand nombre de particules. Par exemple prend un gaz de volume V contenant N particules et soit un sous-système de celui-ci de volume V/k. La statistique montre qu'il y a en moyenne N/k particules avec des fluctuations autour de la valeur moyenne qui vaut Racine de N/k. L'erreur relative tend vers zéro quand N tend vers l'infini. Avec 10 puissance 19 molécules par cm3 l'erreur est tellement faible que l'on atteint la certitude. C'est ainsi que le comportement aléatoire des individus se transforme en certitude pour la collectivité.


    Je connais dans le principe la hierarchie BBGKY (puisqu'utilisée en physique des liquides) et elle ne permet pas a priori d'expliquer l'irreversibilité macroscopique.
    ;
    bien sur que si.

    Lorsqu'on s'interesse à l'ordre 1 de la hierarchie on a un truc du genre

    est une fonctionnelle.
    .
    Non ceci est totalement faux.
    .
    Ce que démontre la hierarchie BBGKY est que:


    Toutes les fonctions de distributions à N particules sont des fonctionnelles de la fonction de distribution à 1 particule. Ceci est vrai pour un temps t>t° ou t° est le temps moyen entre 2 collisions.
    .
    Autrement dit:

    Fn (x1,.......xn,t) = Fn[x1,........xn, F1(x,t)]
    .
    Avec n= 2,3,.......N

    Cela veut dire que toutes les fonctions de distributions sont esclaves de la fonction de distributions à 1 particules et surtout qu'il n' y a pas de dépendance explicite du temps (sauf pour la distribution à 1 particule).
    .
    de là on extrait une équation pour l'évolution de la fonction a 1 particule:

    df/dt + p/m.df/dt = L(x, f)

    avec L(x,f) = Intégrale sur dx2 de V(x-x2).f2 (x,x2,f)
    .



    Ce qu'il se passe c'est que Boltzmann a donné de façon phénomènologique (et un peu had hoc) une forme spécifique à cette fonctionnelle (basée sur l'hypothèse de chaos moléculaire) qui a induit demblé une irreversibilité dans l'equation d'évolution de la distribution à une particule.
    .
    Boltzmann n'a pas écrit à priori l'équation qui porte son nom. Sa démarche est d'une grandeur rigueur intellectuelle. Sa démarche tourne autour d'un bilan de conservation du nombre de particules dans l'espace des phases. Il écrit tout naturellement l'équation d'évolution de la fonction de distribution à 1 particule en supposant que les particules ne sont pas corrélés avant leur chocs: C'est l'hypothèse du chaos moléculaire.
    .
    Autrement dit relativement à la hierarchie BBGKY il a supposé que:

    f2 (x1,x2,t) = f1(x1,t).f1(x2,t)
    .
    Cela veut dire que l'équation de Bolzmann se déduit de la hiéarchie BBGKY avec comme hypothèse supplémentaire sur la forme de la fonctionnelle f2. Ce qui du même coup permet de faire une fermeture très courte puisque l'équation de Botzmann élimine toute forme explicite de la fonction de distribution a 2 particules.
    .
    En résumé l'équation de Boltzmann contiend 2 types d'approximations:
    .
    1- l'esclavage des fonctions de distributions qui interdit de dfécrire quoi que ce soit pour un temps inférieur au temps de collisions.
    .
    2-Une approximation sur la fonction de distributions a 2 particules.

    . Enfin il n'y a aucune contradiction entre les lois réversibles de la mécanique et l'équation de Boltzmann. Cette dernière étant une propriété émergente lié au grans nombre de particules. En termes de théorie des groupes c'est une brisure de symétrie

  14. #13
    gatsu

    Re : J'ai un problème avec les ensembles canoniques

    Citation Envoyé par Anacarsis_47 Voir le message
    pas mal de travaux ont été effectués où sont en cours ce sujet. On montre que la position d'équilibre ne peut être atteinte pour un ensemble trop petit de particules. Typiquement en dessous d'une centaine justement.

    Je t'invite à lire les travaux de P. Gaspard à ce sujet

    par exemple:
    http://scitation.aip.org/getpdf/serv...ps&prog=normal
    Anacarsis
    Ok je vais regarder ça.

    Citation Envoyé par mariposa
    ta phrase est contradictoire en elle-même. Le mécanisme émergent est une propriété nouvelle qui apparait seulement pour un grand nombre de particules. Donc il ne faut pas surtout écrire: "peu importe le nombre de particules".
    Si tu la lis jusqu'au bout elle n'est pas contradictoire. Une propriété émergente (quel que soit le système) n'est appréciable que si on se met à une échelle susceptible de l'observer (ce n'est pas pour rien que j'ai mis cette signature là ). Après le nombre de particules, qui est important c'est sûr, n'est pas forcément pertinent à toutes les échelles.

    Non ceci est totalement faux.
    Ba je serais de curieux de savoir pourquoi. J'ai sous les yeux deux bouquins de référence sur le sujet qui la présente (la hierarchie BBGKY) comme je l'ai dit et je ne l'ai jamais vu écrit comme tu la présentes.
    Tu peux voir aussi ici qu'ils le présentent comme moi
    http://www.springerlink.com/content/j187223063245105/

    Ce que démontre la hierarchie BBGKY est que:
    Toutes les fonctions de distributions à N particules sont des fonctionnelles de la fonction de distribution à 1 particule.
    Je ne sais pas où est ce que tu as vu ça. Ca ressemble d'avantage à des hypothèses en plus de la hierrachie BBGKY pour pouvoir résoudre quelque chose (comme il est montré dans le lien ci dessus).

    Enfin il n'y a aucune contradiction entre les lois réversibles de la mécanique et l'équation de Boltzmann. Cette dernière étant une propriété émergente lié au grans nombre de particules. En termes de théorie des groupes c'est une brisure de symétrie
    Ba quand même une des dynamiques est invariante par renversement du temps et pas l'autre...
    Tu sembles affirmer que l'emergence ne provient que du grand nombre de particules mais ce n'est pas le cas. La résolution en temps et en espace est importante pour observer ce genre de chose. Ceci peut se traduire par l'annulation des fonction de correlation à deux points par exemple comme l'expliquait chaverondier dans ce lien
    http://forums.futura-sciences.com/thread108672.html

    De façon plus générale Balian a également montré que la résolution était importante pour prévoir une irreversibilité
    http://arxiv.org/abs/cond-mat/9907015 (page 14)
    Et le role de la résolution en temps et en espace pour l'évolution de l'entropie de Gibbs comme celle de Boltzmann a été souligné dans
    http://www.amazon.fr/Physique-statis.../dp/2701146062

    P.S.: en relisant ton message il me semble qu'on écrit la même equation au final pour la distribution marginale en notant que pour moi le "d droit" est une dérivée particulaire et que la fonctionnelle correspond à ce que tu écris également.
    "Au fond..la musique si on la prend note par note c'est assez nul". Geluck

  15. #14
    mariposa

    Re : J'ai un problème avec les ensembles canoniques

    Citation Envoyé par gatsu Voir le message
    Ok je vais regarder ça.



    Si tu la lis jusqu'au bout elle n'est pas contradictoire. Une propriété émergente (quel que soit le système) n'est appréciable que si on se met à une échelle susceptible de l'observer (ce n'est pas pour rien que j'ai mis cette signature là ). Après le nombre de particules, qui est important c'est sûr, n'est pas forcément pertinent à toutes les échelles.
    .
    Face a cette phrase je te conseille attentivement de lire les 20 premières pages de LANDAU, physique statistique. La physique statistique c'est l'observation despropriétés d'un ensemble de particules. Il est alors inutile de renvoyer l'observation a une petite échelle de temps ou d'espace car c'est en dehors de la problématique statistique.

    Ba je serais de curieux de savoir pourquoi. J'ai sous les yeux deux bouquins de référence sur le sujet qui la présente (la hierarchie BBGKY) comme je l'ai dit et je ne l'ai jamais vu écrit comme tu la présentes.
    .
    C'est pourquoi j'ai fait un travail de pédagogie qui met en relief l'essentiel de ce qu'est la hiérarchie BBGKY car beaucoup de textes se noient dans les calculs. Il y a eu une fois un étudiant sur Futura qui a montré son cours sur la justification de l'équation de Boltzmann et tout était faux. C'est pas parceque c'est écrit dans un livre que c'est juste. C'est eu l'occasion toute ma vie durant de mesurer les conneries écrites dans les bouquins. En tant que ex-referee de 2 revues scientiques américaines j'ai pu également apprécier la capacité de certains a raconter n'importequoi. Je te conseille vivement de lire les travaux issus de l'école de Bruxelles pour ce qui concerne la thermodynamique classique. En ce qui concerne la physique statistique je te renvois à l'école de Landau/Bogoliuobov et de lire notamment les livres aux éditions de Moscou qui on,t été écrit par leurs collaborateurs. Parmi les livres français très subtilement pédagogique je t'invite à consulter le livre de Hakim(chez Dunod) intitulé introduction àla physique statistique.



    Je ne sais pas où est ce que tu as vu ça. Ca ressemble d'avantage à des hypothèses en plus de la hierrachie BBGKY pour pouvoir résoudre quelque chose (comme il est montré dans le lien ci dessus).
    .
    Ce sont les travaux origaux de Bogolioubov.

    Ba quand même une des dynamiques est invariante par renversement du temps et pas l'autre...
    .
    Cette phrase prouve que que tu ne comprends pas ce qu'est un mécanisme d'émergence. En termes modernes on pourrait dire que la dynamique décrite par l'équation de Boltzmann est une dynamique effective pour les systèmes a grand nombre de particules. De la même façon l'équation de Naviers-Stokes est une équation d'évolution effective pour les systèmes a grande nombre de particules avec en plus une approximation du continu (ce qui n'est pas le cas de l'équation de Boltzmann.

    Tu sembles affirmer que l'emergence ne provient que du grand nombre de particules mais ce n'est pas le cas. La résolution en temps et en espace est importante pour observer ce genre de chose.
    .
    Oui je le réaffirme, c'est un effet du grand nombre et donc d'une échelle d'observation macroscopique ou même mésoscopique.


    Ceci peut se traduire par l'annulation des fonction de correlation à deux points par exemple comme l'expliquait chaverondier dans ce lien
    http://forums.futura-sciences.com/thread108672.html
    .
    u auras du mal a me prouver que cette personne maitrise la physique statistique.


    De façon plus générale Balian a également montré que la résolution était importante pour prévoir une irreversibilité
    .
    Tel que tu l'écris c'est pure tautologie. Je ne crois pas que Balian soit un écrivain en tautologie. Je suis même convaincu du contraire.

    http://arxiv.org/abs/cond-mat/9907015 (page 14)
    Et le role de la résolution en temps et en espace pour l'évolution de l'entropie de Gibbs comme celle de Boltzmann a été souligné dans
    http://www.amazon.fr/Physique-statis.../dp/2701146062
    .
    Evidemment, si tu descends en résolution sufisamment bas tu va trouver par exemple lors du retour à l'équilibre d'un système une fluctuation de diminution d'entropie. Il n'empéche le système augmente son entropie dans son ensemble.

    P.S.: en relisant ton message il me semble qu'on écrit la même equation au final pour la distribution marginale en notant que pour moi le "d droit" est une dérivée particulaire et que la fonctionnelle correspond à ce que tu écris également.
    .
    Ton équation ne fait pas apparaitre le coeur du problème de BBGKY a savoir que les fonctions de distributions à n particules sont des fonctionnelles de la fonction de distributionn à 1 particule. Autrement dit les fonctions de distributions ne dépendent pas explicitement du temps. Si on ne comprends pas çà, on ne comprend rien à la la hierachie BBGKY

  16. #15
    gatsu

    Re : J'ai un problème avec les ensembles canoniques

    Citation Envoyé par mariposa Voir le message
    .
    Face a cette phrase je te conseille attentivement de lire les 20 premières pages de LANDAU, physique statistique. La physique statistique c'est l'observation despropriétés d'un ensemble de particules. Il est alors inutile de renvoyer l'observation a une petite échelle de temps ou d'espace car c'est en dehors de la problématique statistique.
    Un peu facile comme argument je trouve .

    C'est pourquoi j'ai fait un travail de pédagogie qui met en relief l'essentiel de ce qu'est la hiérarchie BBGKY
    J'ai encore une fois devant moi les equations "hierarchiques" et je n'y vois pas ce que tu stipules. C'est peut être équivalent mais alors il y a des quelques étapes de calcul entre les deux.
    En outre
    car beaucoup de textes se noient dans les calculs.
    Les calculs prennent deux pages en partant de l'equation de Liouville ce qui me parait plutot correct...on est très très loin de se noyer.

    I
    l y a eu une fois un étudiant sur Futura qui a montré son cours sur la justification de l'équation de Boltzmann et tout était faux.
    je me méfie très fortement de ce que tu appelles faux.

    C'est pas parceque c'est écrit dans un livre que c'est juste.
    Pas la peine que je lise le Landau alors .

    C'est eu l'occasion toute ma vie durant de mesurer les conneries écrites dans les bouquins.
    ce qui est marrant avec toi c'est qu'apparemment tu n'a s jamais écrit de betise...mon dieu que c'est beau d'être un génie.

    Je te conseille vivement de lire les travaux issus de l'école de Bruxelles pour ce qui concerne la thermodynamique classique.
    Je te remercie mais je suis content de mes connaissances en thermodynamique classique (moins par contre en thermodynamique moderne où je n'y connais pas grand chose même si je fini par voir où tu veux en venir).

    En ce qui concerne la physique statistique je te renvois à l'école de Landau/Bogoliuobov et de lire notamment les livres aux éditions de Moscou qui on,t été écrit par leurs collaborateurs. Parmi les livres français très subtilement pédagogique je t'invite à consulter le livre de Hakim(chez Dunod) intitulé introduction àla physique statistique.
    ce qui est encore plus marrant c'est que tu te fous eperduement de la litterature étudiée par ton interlocuteur ; si ce n'est pas celle que tu as consulté c'est forcément de la mer..e.
    .
    Ce sont les travaux origaux de Bogolioubov.
    Ba je suis content d'avoir aussi vu la constribution d'Yvon, Born et Kirkwood parce que y a aussi ces trois types qui ont participés.

    .
    Cette phrase prouve que que tu ne comprends pas ce qu'est un mécanisme d'émergence. En termes modernes on pourrait dire que la dynamique décrite par l'équation de Boltzmann est une dynamique effective pour les systèmes a grand nombre de particules. De la même façon l'équation de Naviers-Stokes est une équation d'évolution effective pour les systèmes a grande nombre de particules avec en plus une approximation du continu (ce qui n'est pas le cas de l'équation de Boltzmann.
    Les equations effectives s'obtiennent toujours (même si c'est parfois camouflé par un truc appelé chaos moléculaire) en mécanique statistique via un processus de coarse graining et de plus en plus souvent de renormalisation. Lors d'une telle procedure de réduction, on fait expres de perdre des informations sur les degrés de liberté qui ne sont pas pertinents. Ces coarse graining font toujours appel à un cutoff en temps et en espace très important pour faire apparaitre la dynamique effective, il n'y a donc pas que le nombre de particules qui compte.

    Oui je le réaffirme, c'est un effet du grand nombre et donc d'une échelle d'observation macroscopique ou même mésoscopique.
    Le grand nombre de particules qui constitue un systeme n'a rien à voir a priori avec l'échelle d'observation je ne comprends pas ce racourci logique que tu tends à faire systématiquement.

    u auras du mal a me prouver que cette personne maitrise la physique statistique.
    Voilà qui est extremement vexant, j'espere qu'il passera lui aussi dans le coin pour partciper au fil.

    Tel que tu l'écris c'est pure tautologie. Je ne crois pas que Balian soit un écrivain en tautologie. Je suis même convaincu du contraire.
    Je ne vois pas où est la tautologie là dedans.

    Evidemment, si tu descends en résolution sufisamment bas tu va trouver par exemple lors du retour à l'équilibre d'un système une fluctuation de diminution d'entropie. Il n'empéche le système augmente son entropie dans son ensemble.
    Tout dépend le sens que tu donnes au mot "entropie"...y en a quand même un petit paquet de définitions differentes (et celles de l'articles de balian ne sont bien sur pas du tout exhaustives).

    Ton équation ne fait pas apparaitre le coeur du problème de BBGKY a savoir que les fonctions de distributions à n particules sont des fonctionnelles de la fonction de distributionn à 1 particule. Autrement dit les fonctions de distributions ne dépendent pas explicitement du temps. Si on ne comprends pas çà, on ne comprend rien à la la hierachie BBGKY
    Je ne sais pas si je ne comprends rien à la hierarchie BBGKY ou si les auteurs que j'ai lu n'y comprennent rien mais toujours est il qu'ils ont publié sur ce sujet et qu'apparemment ça a plu et pourtant ils ne mentionnaient pas du tout la strategie de cette hierarchie comme tu le fais.
    "Au fond..la musique si on la prend note par note c'est assez nul". Geluck

  17. #16
    invite1c3dc18e

    Re : J'ai un problème avec les ensembles canoniques

    Citation Envoyé par gatsu Voir le message
    Je te remercie mais je suis content de mes connaissances en thermodynamique classique (moins par contre en thermodynamique moderne où je n'y connais pas grand chose même si je fini par voir où tu veux en venir).
    gatsu, j'ai du mal à voir ce que tu appelles thermodynamique moderne et thermo classique... en tout cas si une thermo pourrait être citée comme moderne c'est celle dite "thermo classique de Bruxelles" par mariposa.

    Citation Envoyé par gatsu Voir le message
    Le grand nombre de particules qui constitue un systeme n'a rien à voir a priori avec l'échelle d'observation je ne comprends pas ce racourci logique que tu tends à faire systématiquement.
    le nombre de particules n'a en effet pas de lien à priori avec l'échelle d'observation du système. Cependant, dans les systèmes statistiques c'est le nombre de particules qui régit les propriétés statistiques du système et pas l'échelle d'observation. Pour prendre un exemple frappant: dans le même volume les boules de lotto et les molécules de l'air ambiant sont à la même échelle d'observation mais n'ont pas du tout les mêmes caractéristiques. L'un est maxwellien et l'autre est chaotique. Si tu prends 10^23 boules de lotto dans un volume suffisamment grand pour les contenir et qu'elle aient un libre parcours moyen renormalisé par rapport à la taille des boules équivalent a celui des molécules d'air, dans ce cas les deux systèmes seront totalement équivalents aux interactions près bien sûr.

    Les passages d'échelles d'observation sont équivalents dans le cas où l'on traite des milieux (phases) continus où que l'on a pris comme continus; des liquides par exemple; et encore, à condition qu'il n'y ait pas d'interactions à (trop) longue portée.

    Bon je vais jouer au voyant pour quelques secondes , gatsu tu ne travaillerais pas dans le domaine des phases condensées et ferais de la DFT?

  18. #17
    invite1c3dc18e

    Re : J'ai un problème avec les ensembles canoniques

    Citation Envoyé par gatsu Voir le message
    Tout dépend le sens que tu donnes au mot "entropie"...y en a quand même un petit paquet de définitions differentes (et celles de l'articles de balian ne sont bien sur pas du tout exhaustives).
    tu m'inquiètes là... ce serait bien que l'on aie tous la même définition et qu'il n'y ait pas des définitions différentes suivant les circonstances... ça serait gênant... perso je ne vois l'entropie que telle qu'elle est définie par le deuxième principe et son lien avec les configurations du système via le fait que c'est une grandeur extensive.... faut il aller plus loin pour définir l'entropie???

  19. #18
    chaverondier

    Re : J'ai un problème avec les ensembles canoniques

    Citation Envoyé par mariposa Voir le message
    Enfin il n'y a aucune contradiction entre les lois réversibles de la mécanique et l'équation de Boltzmann.
    Mouais, ça demande quand même un peu d'explication (et cette explication n'est pas si largement connue et si bien comprise que ça). La réversibilité des évolutions (cad, en gros, le renversement (sans déviation) de l'évolution des systèmes isolés par renversement des vitesses d'évolution de toutes les grandeurs d'état du système) s'applique à des systèmes censés être vraiment isolés.

    Au contraire, la croissance de l'entropie des système "isolés" (violant cette réversibilité) s'applique à des systèmes qui, en fait, ne sont pas vraiment parfaitement isolés. Dans le cas inverse comme l'a, à juste titre, fait remarquer Loschmidt en son temps, il y aurait effectivement contradiction entre la réversibilité des lois fondamentales de la physique et l'irréversibilité constatée de l'évolution des systèmes physiques "isolés" observés à notre échelle (et ce, indépendemment de toute considération de nombre de particules en jeu).
    Citation Envoyé par mariposa Voir le message
    Cette dernière [l'équation de Boltzmann] étant une propriété émergente liée au grand nombre de particules.
    En fait, de façon plus précise, l'émergence de cette propriété découle du fait que l'information de corrélation induite par les chocs entre particules du système "isolé" se diffuse très rapidement dans l'environnement du système (donc loin de l'observateur implicitement impliqué dans la notion d'entropie de Boltzmann et, donc, implicitement impliqué aussi dans le théorème H de Boltzmann).

    A cause de ce phénomène de fuite d'information de corrélation dans l'environnement, du au fait qu'en pratique aucun système n'est jamais parfaitement isolé de son environnement, la fonction de distribution à 2 particules d'un gaz parfait "isolé" (objet du théorème H de Boltzmann) tend très rapidement (c'est à dire en un temps petit devant le temps de libre parcours moyen) vers le produit de ses distributions marginales. Ces dernières se trouvent être égales à la fonction de distribution à une particule du gaz parfait "isolé".

    Cette fuite d'information loin du système "isolé" est modélisée (en fait) par l'hypothèse du chaos moléculaire (alors qu'initialement ce n'était pas du tout le sens physique attribué par Boltzmann à cette hypothèse). En définitive, l'absence de contradiction entre la croissance (observée) de l'entropie des systèmes "isolés" et l'évolution (théoriquement) isentropique des systèmes isolés régis par une dynamique hamiltonnienne résulte du fait que, en pratique, les systèmes "isolés" ne le sont jamais vraiment (cf la remarque de Borel sur l'effet d'une perturbation du champ de gravitationnel, d'un ordre de grandeur égal à celui d'un sucre placé à proximité de Sirius, sur la dynamique d'évolution de l'état microphysique d'un gaz évoluant, à proximité de la terre, dans une enceinte censée l'isoler de son environnement).

  20. #19
    gatsu

    Re : J'ai un problème avec les ensembles canoniques

    Citation Envoyé par Anacarsis_47 Voir le message
    gatsu, j'ai du mal à voir ce que tu appelles thermodynamique moderne et thermo classique... en tout cas si une thermo pourrait être citée comme moderne c'est celle dite "thermo classique de Bruxelles" par mariposa.
    La thermodynamique "classique" ou thermostatique comme on dit maintenant est la science qui s'est développée au XIXe et qui décrit les états finaux et intiaux de systèmes à l'équilibre avant et après une transformation. Elle ne réfère donc qu'à des grandeurs d'équilibre et ne parle que de variables d'états (d'équilibre) et de fonctions d'états. Dans ce contexte les mots irreversibles, réversibles et quasi-statique pour ne citer qu'eux ont un sens bien précis et clair.
    En thermodynamique moderne initiée notamment par Prigogine, on s'intéresse à l'évolution de variables d'état et de fonctions d'état hors d'équilibre. Le temps intervient explicitement dans les equations par exemple etc...
    Le fait est qu'il semble qu'il y ait enormément d'écoles differentes de thermodynamique moderne et l'école de Prigogine en fait partie. Ceci est en partie dû, semblerait il, au fait qu'il n'y a pas de consensus sur les défintions des variables et fonctions d'états lorsqu'on est hors equilibre (je suis encore super novice dans ce domaine là mais par contre la thermo standard je pense que ça va et je ne pense pas qu'elles soient contradictoires).

    le nombre de particules n'a en effet pas de lien à priori avec l'échelle d'observation du système. Cependant, dans les systèmes statistiques c'est le nombre de particules qui régit les propriétés statistiques du système et pas l'échelle d'observation. Pour prendre un exemple frappant: dans le même volume les boules de lotto et les molécules de l'air ambiant sont à la même échelle d'observation mais n'ont pas du tout les mêmes caractéristiques. L'un est maxwellien et l'autre est chaotique. Si tu prends 10^23 boules de lotto dans un volume suffisamment grand pour les contenir et qu'elle aient un libre parcours moyen renormalisé par rapport à la taille des boules équivalent a celui des molécules d'air, dans ce cas les deux systèmes seront totalement équivalents aux interactions près bien sûr.
    Lorsqu'on a une équation dévolution fondamentalement invariante par renversement du temps, elle ne va pas devenir non invariante par renversement du temps seulement parce qu'il y a beaucoup de particules. Il faut s'intéresser à des macrovariables ("coarse grainée") qui ne contiennent donc pas par défintion toute l'information du système pour qu'une telle chose se produise, c'est tout ce que je dis.
    Par ailleurs, on peut utiliser l'outil statistique a priori quelque soit le nombre de particules mais les fluctuations seront grandes "c'est tout".

    Bon je vais jouer au voyant pour quelques secondes , gatsu tu ne travaillerais pas dans le domaine des phases condensées et ferais de la DFT?
    Ouah t'es devin ?

    tu m'inquiètes là... ce serait bien que l'on aie tous la même définition et qu'il n'y ait pas des définitions différentes suivant les circonstances... ça serait gênant... perso je ne vois l'entropie que telle qu'elle est définie par le deuxième principe et son lien avec les configurations du système via le fait que c'est une grandeur extensive.... faut il aller plus loin pour définir l'entropie???
    Ba par exemple prends un gaz de N particules, d'energie totale E dans une boite de volume V. Tu peux déjà définir l'entropie de Gibbs quelque soit l'état supposé du système (c'est ça que je tiens à souligner dans mon premier message)
    Ba tu peux vérifier que ça va jamais bouger alors que je ne sais pas si le système est à l'équilibre ou pas a priori.
    Ensuite tu peux utiliser l'entropie de Boltzmann :

    qui bien entendu va pouvoir varier dans le temps si on se met dans une "bonne" condition initiale.
    Ensuite pour les systèmes avec des intéractions longue portée on peut définir une entropie à partir du principe des larges déviations, sans compter après les entropies associées à la dynamique du système etc...
    J'ai mis en lien un papier de balian sur ce sujet, il est intéresssant sans déconner.
    "Au fond..la musique si on la prend note par note c'est assez nul". Geluck

  21. #20
    invite1c3dc18e

    Re : J'ai un problème avec les ensembles canoniques

    Citation Envoyé par gatsu Voir le message
    Ouah t'es devin ?
    peut-être

    Citation Envoyé par gatsu Voir le message
    Ba par exemple prends un gaz de N particules, d'energie totale E dans une boite de volume V. Tu peux déjà définir l'entropie de Gibbs quelque soit l'état supposé du système (c'est ça que je tiens à souligner dans mon premier message)
    Ba tu peux vérifier que ça va jamais bouger alors que je ne sais pas si le système est à l'équilibre ou pas a priori.
    Ensuite tu peux utiliser l'entropie de Boltzmann :

    qui bien entendu va pouvoir varier dans le temps si on se met dans une "bonne" condition initiale.
    Ensuite pour les systèmes avec des intéractions longue portée on peut définir une entropie à partir du principe des larges déviations, sans compter après les entropies associées à la dynamique du système etc...
    J'ai mis en lien un papier de balian sur ce sujet, il est intéresssant sans déconner.
    l'entropie de Gibbs est définie comme tu l'écris pour un état d'équilibre.
    Ta deuxième expression par contre est phénoménologique si j'interprète bien tes notations non définies . Tu y as introduit le temps et de plus partitionné l'espace en boîtes à l'intérieur desquelles le système est supposé être dans un équilibre local...

  22. #21
    gatsu

    Re : J'ai un problème avec les ensembles canoniques

    Citation Envoyé par Anacarsis_47 Voir le message
    peut-être
    nan mais sans dec si t'as trouvé ça seulement avec ma manière d'argumenter t'es trop fort .
    l'entropie de Gibbs est définie comme tu l'écris pour un état d'équilibre.
    Je crois que tu n'a pas bien compris le but premier de ce fil. Justement la seule entropie de Gibbs qu'on puisse faire avec la connaissance de E,V,N est celle que j'ai écrite que cela soit en passant par la théorie ergodique en générale ou par le principe d'indifference. Comme E,V et N sont des constantes au cours du temps (car dynamique hamiltonienne supposée) il n'y a aucun moyen de savoir si le système est à l'équilibre ou non. C'est précisément cette contradiction de vocabulaire (selon moi) que je souhaite soulever dans ce fil.

    Ta deuxième expression par contre est phénoménologique si j'interprète bien tes notations non définies
    Tu y as introduit le temps et de plus partitionné l'espace en boîtes à l'intérieur desquelles le système est supposé être dans un équilibre local...
    Ma deuxième expression n'est pas plus phénomènologique que la première. C'est juste que comme elle est construite differemment, elle ne conduit pas au même résultat.
    En effet, pour tout microétat du système à un instant donné tu peux faire une partition linéaire de l'espace des phases à une particule et compter ensuite (en principe) combien de particules il y a dans chaque cellule. De ce comptage, on en déduit une distribution de probabilité qui, vu qu'elle est prise pour n'importe quelle point d'une trajectoire inconnue, va dépendre du point de l'espace et du temps également (puisqu'on est à un instant donné). Cette distribution est notée classiquement .
    A partir de cette distribution, rien ne ,nous empèche d'essayer de savoir quelle serait le manque d'information qu'on aurait sur le système si on ne connaissait que les , on en déduit alors l'entropie de Boltzmann (soit ).

    Ce que je dis c'est que l'ensemble des est un ensemble de macrovariables pertinent pour décrire ce qu'est le non equilibre (même si il y a equilibre local) et par corrolaire ce qu'est l'équilibre. A contrario, les macrovariables E,V et N étant des constantes du mouvement, elles ne peuvent pas constituer un critère permettant de savoir si le système est à l'équilibre ou non. Aussi, c'est pourquoi ça me semble falacieux de parler d'entropie de Gibbs à l'equilibre si on a seulement la connaissance de E,V et N puisque c'est la seule qu'on puisse construire en fait en ayant ces informations sur le système.
    "Au fond..la musique si on la prend note par note c'est assez nul". Geluck

  23. #22
    invite1c3dc18e

    Re : J'ai un problème avec les ensembles canoniques

    Citation Envoyé par gatsu Voir le message
    nan mais sans dec si t'as trouvé ça seulement avec ma manière d'argumenter t'es trop fort .
    oui c'est exactement comme ça, il y a certains détails qui n'ont pas trompé

    Citation Envoyé par gatsu Voir le message
    Cette distribution est notée classiquement .
    mais oui... autant pour moi... en fait j'ai utilisé cette notation pendant très longtemps pour la densité locale de particules d'où ma confusion... sorry

    en effet j'ai un peu de mal à voir le but premier du fil... faudrais que je relise tout...

  24. #23
    mariposa

    Re : J'ai un problème avec les ensembles canoniques

    Citation Envoyé par chaverondier Voir le message
    Mouais, ça demande quand même un peu d'explication (et cette explication n'est pas si largement connue et si bien comprise que ça). La réversibilité des évolutions (cad, en gros, le renversement (sans déviation) de l'évolution des systèmes isolés par renversement des vitesses d'évolution de toutes les grandeurs d'état du système) s'applique à des systèmes censés être vraiment isolés.

    Au contraire, la croissance de l'entropie des système "isolés" (violant cette réversibilité) s'applique à des systèmes qui, en fait, ne sont pas vraiment parfaitement isolés.
    Bonjour,

    Tu as déjà écris cela plusieurs fois. Le problème ne vient du fait que le système ne soit pas parfaitement isolé.

    Exemple simple: Un ensemble de N particules dans la moitié gauche d'une enceinte parfaitement isolée va évoluer en 1.mS à peine et il y aura une répartition stationnaire moitié-moitié. La dynamique des particules obéit à la fois à la mécanique classique et à l'évolution temporellement fléchée vers un équilibre moitié-moitié. Il y a un phénomène d'émergence lié au très grand nombre de particules.
    .
    Ceci est intuitif, reste à le démontrer proprement (ce qui renvoie à la hiérarchie BBGKY et autres démonstration).

  25. #24
    gatsu

    Re : J'ai un problème avec les ensembles canoniques

    Citation Envoyé par Anacarsis_47 Voir le message
    oui c'est exactement comme ça, il y a certains détails qui n'ont pas trompé
    Comme quoi par exemple ? J'essaie pourtant de rester "neutre" dans mon vocabulaire...flute .

    en effet j'ai un peu de mal à voir le but premier du fil... faudrais que je relise tout...
    Pas besoin de tout lire je pense. J'ai normalement résumé mon interrogation dans la fin de mon dernier message et dans le premier message de ce fil après je n'ai fait que répondre aux objections "classiques" de mariposa qui m'invite gentillement à regarder un cours de méca. stat. (même pas d'ailleurs, l'intro. du bouqin seulement).
    La raison de mon interrogation réside peut être dans des lacunes dans mes connaissances mais je ne pense pas...

    P.S : le lien vers la discussion que j'ai donné dans mon tout premier message est a priori plus constructif que l'enguelade bourrée d'affirmations peremptoires qui a partiellement lieu ici pour l'instant.
    "Au fond..la musique si on la prend note par note c'est assez nul". Geluck

  26. #25
    chaverondier

    Re : J'ai un problème avec les ensembles canoniques

    Citation Envoyé par mariposa Voir le message
    Tu as déjà écris cela plusieurs fois. Le problème ne vient du fait que le système ne soit pas parfaitement isolé.
    Si, si.
    Citation Envoyé par mariposa Voir le message
    Exemple simple: Un ensemble de N particules dans la moitié gauche d'une enceinte parfaitement isolée va évoluer en 1.mS à peine et il y aura une répartition stationnaire moitié-moitié.
    C'est effectivement un exemple classique et bien adapté à la discussion (voir plus loin les détails relatifs à cet exemple)
    Citation Envoyé par mariposa Voir le message
    La dynamique des particules obéit à la fois à la mécanique classique et à l'évolution temporellement fléchée vers un équilibre moitié-moitié. Il y a un phénomène d'émergence lié au très grand nombre de particules.
    Non. En raison de l'impossibilité pratique d'isoler le système considéré de son environnement, l'irréversibilité de l'évolution des systèmes se manifeste même si leur nombre de particules n'est pas très grand.
    Citation Envoyé par mariposa Voir le message
    Ceci est intuitif, reste à le démontrer proprement (ce qui renvoie à la hiérarchie BBGKY et autres démonstrations).
    Pour démontrer proprement le caractère pertinent (eu égard à la limitation d'accès à l'information des observateurs que nous sommes) de la coupure de la hiérarchie BBGKY au niveau le plus grossier (modélisation de l'état du gaz dans son espace de phase à une seule particule) il faut faire intervenir la fuite d'information loin de l'observateur (modélisée, dans le cas des gaz parfaits, par l'hypothèse du chaos moléculaire bien que la signification physique de cette hypothèse, en terme de fuite d'information dans le milieu extérieur au gaz, ne soit pas très largement connue).

    On peut préciser plus en détail cette question délicate. Si le système formé du gaz occupant initialement la moitié d'un réservoir était réellement parfaitement isolé (pas de fuite d'information hors de portée de l'observateur) alors :

    1/ en renversant les vitesses de toutes les particules du gaz elles réoccuperaient la moitié du réservoir qu'elles ont quitté (car les lois fondamentales de la physique sont (presque) time symetric). Elles reprendraient très exactement les positions et vitesses qui étaient les leurs dans la situation initiale où elles n'occupaient qu'une moitié du réservoir (on retrouve dans cet effet l'objection dite de réversibilité faite à Boltzmann par Loschmidt)

    2/ en attendant "suffisamment longtemps" (certes un nombre respectable de fois l'âge supposé de l'univers) le système repasserait aussi près qu'on le souhaite de son état initial (on retrouve dans cet effet l'objection formulée par Zermelo reposant sur le théorème de récurrence de Poincaré).

    La réponse à ces objections est la suivante : l'irréversibilité des évolutions existe bien (au moins d'une certaine façon puisqu'on l'observe) mais elle repose sur l'impossibilité pratique d'isoler un système de son environnement et sur l'impossibilité, pour un observateur, de conserver indéfiniment toute l'information sur un système donné et sur tout ce avec quoi il interagit. Sans cette fuite d'information loin de l'observateur, il n'y a
    • plus de croissance de l'entropie des systèmes "isolés",
    • plus de possibilité d'enregistrer de l'information,
    • plus d'irréversibilité,
    • plus d'écoulement du temps,
    • plus de non localité quantique,
    • plus d'indéterminisme quantique.

    cf aussi l'hypothèse du temps thermique proposée par C. Rovelli et A. Connes pour un développement de ce type de réflexion impliquant la mécanique quantique.

  27. #26
    mariposa

    Re : J'ai un problème avec les ensembles canoniques

    Citation Envoyé par chaverondier Voir le message
    Si, si.
    C'est effectivement un exemple classique et bien adapté à la discussion (voir plus loin les détails relatifs à cet exemple)
    Non. En raison de l'impossibilité pratique d'isoler le système considéré de son environnement, l'irréversibilité de l'évolution des systèmes se manifeste même si leur nombre de particules n'est pas très grand.
    .
    Mais où as-tu donc dénicher çà?. Le principe de la thermodynamique est de raisonner sur un système isolé: Ensemble microcanonique. Quan un système n'est pas isolé on considère le système et le milieu extérieur formant ensemble un système isolé c'est le principe d'un ensemble canonique.

    Pour démontrer proprement le caractère pertinent (eu égard à la limitation d'accès à l'information des observateurs que nous sommes) de la coupure de la hiérarchie BBGKY au niveau le plus grossier (modélisation de l'état du gaz dans son espace de phase à une seule particule)
    J'ai expliqué plus haut en quioi consiste physiquement et mathématiquement ce qu'est la hierarchie BBGKY. il n'est pas du tout question de coupure maus de fermeture d'un système d'équations (grace à l'approximation des modes esclaves ( ou adiabatique comme on voudra)

    il faut faire intervenir la fuite d'information loin de l'observateur (modélisée, dans le cas des gaz parfaits, par l'hypothèse du chaos moléculaire bien que la signification physique de cette hypothèse, en terme de fuite d'information dans le milieu extérieur au gaz, ne soit pas très largement connue).
    .
    L'hypothèse du chaos moléculaire ne concerne pas les gaz parfaits, bien au contraire. Elle consiste à supposer que les corrélations de paires entre particules ne controle pas la cinétique de collisions. Cette hypothèse hautement raisonable permet techniquement de calculer les termes de collisions aux second membre de l'équation de Boltzmann.


    1/ en renversant les vitesses de toutes les particules du gaz elles réoccuperaient la moitié du réservoir qu'elles ont quitté (car les lois fondamentales de la physique sont (presque) time symetric). Elles reprendraient très exactement les positions et vitesses qui étaient les leurs dans la situation initiale où elles n'occupaient qu'une moitié du réservoir (on retrouve dans cet effet l'objection dite de réversibilité faite à Boltzmann par Loschmidt)
    .
    C'est vrai en principe, sauf que c'est impossible a réaliser pratiquement.

    2/ en attendant "suffisamment longtemps" (certes un nombre respectable de fois l'âge supposé de l'univers) le système repasserait aussi près qu'on le souhaite de son état initial (on retrouve dans cet effet l'objection formulée par Zermelo reposant sur le théorème de récurrence de Poincaré).
    .
    Là aussi c'est une condition totalement éloigné du réel. C'est juste pour le fun intellectuel.

    La réponse à ces objections est la suivante : l'irréversibilité des évolutions existe bien (au moins d'une certaine façon puisqu'on l'observe) mais elle repose sur l'impossibilité pratique d'isoler un système de son environnement et sur l'impossibilité, pour un observateur, de conserver indéfiniment toute l'information sur un système donné et sur tout ce avec quoi il interagit. Sans cette fuite d'information loin de l'observateur, il n'y a
    • plus de croissance de l'entropie des systèmes "isolés",
    • plus de possibilité d'enregistrer de l'information,
    • plus d'irréversibilité,
    • plus d'écoulement du temps,
    • plus de non localité quantique,
    • plus d'indéterminisme quantique.

    cf aussi l'hypothèse du temps thermique proposée par C. Rovelli et A. Connes pour un développement de ce type de réflexion impliquant la mécanique quantique.
    .
    Toutes ces considérations sur la fuite d'information n'a rien à voir avec la pratique de la thermodynamique. C'est l'impressuin que tu fais une maillonazise avec pleins de considérations qui ne sont pas sur le même plan.

  28. #27
    gatsu

    Re : J'ai un problème avec les ensembles canoniques

    Citation Envoyé par chaverondier Voir le message
    On peut préciser plus en détail cette question délicate. Si le système formé du gaz occupant initialement la moitié d'un réservoir était réellement parfaitement isolé (pas de fuite d'information hors de portée de l'observateur) alors :

    1/ en renversant les vitesses de toutes les particules du gaz ....du réservoir (on retrouve dans cet effet l'objection dite de réversibilité faite à Boltzmann par Loschmidt)

    2/ en attendant "suffisamment longtemps" (certes un nombre respectable de fois l'âge supposé de l'univers) le système repasserait aussi près qu'on le souhaite de son état initial (on retrouve dans cet effet l'objection formulée par Zermelo reposant sur le théorème de récurrence de Poincaré).

    La réponse à ces objections est la suivante : l'irréversibilité des évolutions existe bien (au moins d'une certaine façon puisqu'on l'observe) mais elle repose sur l'impossibilité pratique d'isoler un système de son environnement
    Ce n'est pas la réponse "conventionnelle" c'est important de le souligner. Boltzmann a, il me semble, bien répondu à ces objections en utilisant l'interprétation statistique du théorème H et en plus en ce qui concerne l'objection de Loschmidt, pas besoin d'invoquer une fuite d'information, c'est tout bonnement impossible, en pratique, d'inverser la vitesse des particules pour qu'elles retournent vers le bon macroétat et encore moins vers le microétat de départ.
    Cette idée que l'explication de la méca. stat. (dans son ensemble) vient fondamentalement de l'impossibilité pratique d'isoler correctement un système fait partie d'un courant de pensée dans la communauté des statisticiens (Huang notamment a beaucoup revendiqué cette idée pour rationnaliser les mélanges statistiques en MQ) mais elle est loin de faire l'unanimité. Je n'ai pour ma part jamais vu de calcul ni de simulations qui montraient que cette hypothèse marchait au delà du statut de "raisonnable".
    De toute façon, de nombreux travaux sur des sytèmes hamiltoniens (avec une approche plus théorie cinétique des gaz) ont été faits et conduisent bel et bien à l'irreversibilité, à la relaxation vers l'équilibre etc...et sans faire intervenir l'exterieur.
    "Au fond..la musique si on la prend note par note c'est assez nul". Geluck

  29. #28
    mariposa

    Re : J'ai un problème avec les ensembles canoniques

    Citation Envoyé par gatsu Voir le message
    Ce n'est pas la réponse "conventionnelle" c'est important de le souligner. Boltzmann a, il me semble, bien répondu à ces objections en utilisant l'interprétation statistique du théorème H et en plus en ce qui concerne l'objection de Loschmidt, pas besoin d'invoquer une fuite d'information, c'est tout bonnement impossible, en pratique, d'inverser la vitesse des particules pour qu'elles retournent vers le bon macroétat et encore moins vers le microétat de départ.
    Cette idée que l'explication de la méca. stat. (dans son ensemble) vient fondamentalement de l'impossibilité pratique d'isoler correctement un système fait partie d'un courant de pensée dans la communauté des statisticiens (Huang notamment a beaucoup revendiqué cette idée pour rationnaliser les mélanges statistiques en MQ) mais elle est loin de faire l'unanimité. Je n'ai pour ma part jamais vu de calcul ni de simulations qui montraient que cette hypothèse marchait au delà du statut de "raisonnable".
    De toute façon, de nombreux travaux sur des systèmes hamiltoniens (avec une approche plus théorie cinétique des gaz) ont été faits et conduisent bel et bien à l'irreversibilité, à la relaxation vers l'équilibre etc...et sans faire intervenir l'exterieur.
    .
    Bonjour,
    .
    Merci pour cette information concernant ce courant de pensée concernant les systèmes isolés pas vraiment isolés. A l'évidence cela ne tiend pas la route et je m'étonne même que des gens probablement "raisonnables" aient pu se fourvoyer dans de telles voies. Si nécessaire je reviendrais sur cette question.
    .
    Si tu pouvais m'indiquer un pdf sur cette question, ce serait bien afin que j' établisse une critique honnète et constructive et élaborée sur cette question.
    .
    Merci par anticipation.
    .

  30. #29
    gatsu

    Re : J'ai un problème avec les ensembles canoniques

    Citation Envoyé par mariposa Voir le message
    .
    Bonjour,
    .
    Merci pour cette information concernant ce courant de pensée concernant les systèmes isolés pas vraiment isolés. A l'évidence cela ne tiend pas la route et je m'étonne même que des gens probablement "raisonnables" aient pu se fourvoyer dans de telles voies. Si nécessaire je reviendrais sur cette question.
    .
    Si tu pouvais m'indiquer un pdf sur cette question, ce serait bien afin que j' établisse une critique honnète et constructive et élaborée sur cette question.
    .
    Merci par anticipation.
    .
    Malheureusement je n'ai pas de pdf a te filer sur le sujet. La seule chose que je sais c'est que Huang soutient dans son bouqin de méca. stat. ce genre de théorie mais axé sur la statistique des systèmes quantiques (je pense que Huang a "raison" dans le sens où la décohérence vient fondamentalement du fait qu'un système quantique macroscopique ne peut jamais être totalement isolé de l'extérieur mais de là à imaginer que l'irreversibilité thermodynamique vient de là...c'est déjà plus douteux) c'est pourquoi (Huang n'étant pas un brank) je me suis permis d'utiliser le terme "courant de pensée" d'autant que j'ai rencontré ses idées ailleurs mais je me rappelle plus où.
    Toujours est il que ce point de vue est tellement connu que je n'arrive pas à trouver de référence en ligne sur ce sujet.
    Il faut peut être attendre que Chaverondier nous file des liens, si il en connait, sur ce point de vue pour que moi aussi je me fasse une meilleure idée de quoi il en retourne (mais je ne pense pas que ça me fera changer d'avis...).
    "Au fond..la musique si on la prend note par note c'est assez nul". Geluck

  31. #30
    gatsu

    Re : J'ai un problème avec les ensembles canoniques

    En bref je viens de me rendre compte ni plus ni moins que je remettais en cause ce qui est pour certains le postulat fondamental de la mécanique statistique (comme l'avait souligné mariposa) :

    "Pour un système isolé à l'équilibre, tous les microétats sont équiprobables"

    Je trouve que dans cette formulation (pas suffisamment explicite) le terme équilibre est abusif et non relevant pour utiliser le postulat d'équiprobabilité des microétats.
    Ca ne me gène pas plus que ça de ne pas être d'accord avec ce postulat puisque je m'intéresse aux fondements de la MS et que je n'ai pas de raison a priori d'accepter ce postulat sans justification (que j'estime pouvoir exister).
    Par ailleurs le fait que ça marche ne constitue pas un argument valable en principe (mais seulement en pratique) puisqu'à la limite thermo la mesure du volume de l'espace des phases (i.e. microcanonique) correspondant à des microétats hors equilibre tend vers zero.
    "Au fond..la musique si on la prend note par note c'est assez nul". Geluck

Page 1 sur 2 1 DernièreDernière

Discussions similaires

  1. Problème sur les ensembles de points
    Par invite143e4ff1 dans le forum Mathématiques du collège et du lycée
    Réponses: 19
    Dernier message: 09/03/2008, 20h20
  2. J'ai un problème avec les Reactions chimiques
    Par invite8ca46a43 dans le forum Chimie
    Réponses: 1
    Dernier message: 16/12/2007, 10h12
  3. j'ai un problème avec les Séismes!!!
    Par invitef186a3e7 dans le forum Géologie et Catastrophes naturelles
    Réponses: 1
    Dernier message: 23/11/2007, 06h33
  4. problème sur les ensembles et relations (niveau MPSI)
    Par invite56f88dc9 dans le forum Mathématiques du supérieur
    Réponses: 15
    Dernier message: 28/09/2006, 21h40
  5. J'ai un problème avec les graphes (décomposition des flots)
    Par aiolia dans le forum Mathématiques du supérieur
    Réponses: 19
    Dernier message: 20/03/2005, 01h45